Solve these linear systems using elimination <3
1. 4x-3y=8
5x-2y=11

2. -2x-5y=9
3x+11y=4

3. 7x-6y=-1
5x-4y=1

Answers

Answer 1

Answer:

1.x=3/4y + 2

Step-by-step explanation:

Add 3y to both sides then after youdidnt that divide both sides by 4 Thereyougo the answer.

i have school myself so i only did one.


Related Questions

1. The graph of a quadratic function is called a(an)
Copy and complete the table of values for the function.
2.
y=-1/3 x²

Answers

The graph of a quadratic function is called a parabola

The complete table is

x = -6, -3, 0, 3 6

y = -12  -3 0 -3 -12

How to complete the table of values?

From the question, we have the following equation that can be used in our computation:

y=-1/3 x²

From the table of values , we have the following x values

x = -6, -3, 0, 3 and 6

Substitute x = -6, -3, 0, 3 and 6 in y=-1/3 x²

So, we have the following representation

y = -1/3 (-6)² = -12

y = -1/3 (-3)² = -3

y = -1/3 (0)² = 0

y = -1/3 (3)² = -3

y = -1/3 (6)² = -12

So, the complete table of values is

x = -6, -3, 0, 3 6

y = -12  -3 0 -3 -12

Read more about quadratic function at

https://brainly.com/question/25841119

#SPJ1

The measures of the angles of a triangle are shown in the figure below. Solve for x.
to
76⁰
41

Answers

The measures of the angles of a triangle are 76⁰, 41⁰, and 63⁰.

What is Angle Sum Property?

The sum of all angles of a triangle is equal to the angle of a straight line i.e. 180°. If we have a triangle ABC, then the Sum of angles A , B, and angle C will be 180 ° and the value of the exterior angle is equal to the sum of two interior opposite angle.

We have,

The measures of the angles of a triangle are:

x,

76⁰,

41⁰.

so, we have to calculate the value of the x⁰:

The sum of all the angles in the triangle is 180⁰

so,

to calculate the measure of x put the all values in equation such that using the above rule:

x + 76⁰ + 41⁰ = 180⁰

x = 180⁰ -  76⁰ - 41⁰

x =  63⁰

Hence, the measures of the angles of a triangle are 76⁰, 41⁰, and 63⁰.

To know more about the Angle sum property visit,

brainly.com/question/21099419

#SPJ1

coordinate plane notes pratice

help pleaseeee tyyyvm :)

Answers

The graph in which point are plotted is given below and Point D and F lie on Y axis and X axis respectively other lie on the different quadrant

What does a math quadrant mean?

The term "quadrant" refers to each of the four divisions of the coordinate plane. Furthermore, when we speak of the sections, we also mean the sections as the coordinate axes split them. This is where the x-axis is, and this is where the y-axis is on the up-down axis. On a coordinate plane, it divides the space into four parts, as you can see.

Point A (-5,4)

Lies in 2 quadrant of the graph

Point B (-3,-2)

Lies in 3 quadrant of the graph

Point C (4,6)

Lies in 1 quadrant of the graph

Point D (0,-4)

Lies on negative y axis of the graph

Point E (2,-1)

Lies in 4 quadrant of the graph

Point F (1,0)

Lies on positive x axis of the graph

Learn more about quadrant

https://brainly.com/question/25038683

#SPJ1

use the function f and the given real number a to find (f -1)'(a). (hint: see example 5. if an answer does not exist, enter dne.) f(x)

Answers

The function y= f(x)= [tex]x^{3}+3x-1[/tex] and a= -5 the real number is [tex]\frac{1}{6}[/tex].

y= f(x)= [tex]x^{3}+3x-1[/tex] and a= -5.

The slope of inverse functions are reciprocals at their corresponding points that is,

[tex]f^{-1}'(a)=\frac{1}{f'(b)}[/tex]

Where, [tex]f^{-1}'(a) = b[/tex] and f(b) = a

Now, determine the value of b for a= -5 using f(x)=[tex]x^{3}+3x-1[/tex]

f(b) = a

[tex]b^{3}+3b-1=-5[/tex]

[tex]b^{3}+3b= -4[/tex]

[tex]b = -1[/tex]

Therefore, [tex]f^{-1}'(a) = \frac{1}{f'(b)}[/tex]

[tex]f^{-1}'(-5)=\frac{1}{f'(-1)}[/tex]

Now, find f'(x) and evaluate it at

x= -1

f(x)=[tex]x^{3}+3x-1[/tex]

[tex]f'(x)= 3x^{2}+3[/tex]

f'(x)= [tex]3(x^{2} +1)[/tex]

then, f'(-1)= 3(1+1) = 6

Therefore, [tex]f^{-1}(-5)=\frac{1}{f^{-1}(-1)}=\frac{1}{6}[/tex].

For such more questions about function f:

https://brainly.com/question/29092274

#SPJ4

pls whats the anwer im stuck 4+5(x-1)=34

Answers

Answer: 7

Step-by-step explanation:

To solve this equation, we first need to use the order of operations to simplify the expression on the left-hand side of the equation. The order of operations tells us that we should perform the operations within parentheses first, followed by any exponents, then multiplication and division (from left to right), and finally addition and subtraction (from left to right).

In this case, we have a set of parentheses with the term 5(x-1) inside. This means we need to calculate the value of 5(x-1) first. We can do this by multiplying 5 and (x-1), which gives us 5x-5.

Next, we need to add 4 to 5x-5 to get the final expression on the left-hand side of the equation. This gives us 4+5x-5, which simplifies to 5x-1.

Now we have the equation 5x-1=34. To solve this equation for the value of x, we need to isolate the x term on one side of the equation by moving all the other terms to the other side. We can do this by adding 1 to both sides of the equation, which gives us 5x = 35.

Finally, we can divide both sides of the equation by 5 to find the value of x. This gives us x = 35/5 = 7. Therefore, the value of x that makes the equation true is 7.

Answer:7

Step-by-step explanation:

4+5(x-1)=34

Distribute: 4+5(x - 1) = 34

                   4+5x - 5=34

Subtract the numbers:  4 + 5x - 5=34

                                        -1 + 5x =34

Rearrange terms:    -1 + 5x =34

                                    5x - 1 = 34

Solution: x=7

Hope this helps!!

Which of the following research situations would be most likely to use an independent-measures design?
a) examine the development of vocabulary as a group of children mature from age 2 to age 3
b) examine the long-term effectiveness of a stop-smoking treatment by interviewing subjects 2 months and 7 months after the treatment ends
c) compare the mathematics skills for 9th grade boys versus 9th grade girls*
d) compare the blood-pressure readings before medication and after medication for a group a patients with high blood pressure

Answers

Option C, The research situation that would be most likely to use an independent-measures design is to compare the mathematics skills of 9th-grade boys versus 9th-grade girls.

An independent-measures design is used when there are two or more groups that are being compared and each individual only belongs to one group. In this research situation, the two groups being compared are 9th-grade boys and 9th-grade girls. Each individual in the study belongs to one group or the other, and they are not part of both groups. This makes it an ideal situation for using an independent-measures design.

Option a) examine the development of vocabulary as a group of children mature from age 2 to age 3 would likely use a within-subjects design, as the same group of children is being measured at two different time points.

Option b) Examining the long-term effectiveness of a stop-smoking treatment by interviewing subjects 2 months and 7 months after the treatment ends would also likely use a within-subjects design, as the same group of subjects is being measured at two different time points.

Option d) compare the blood-pressure readings before medication and after medication for a group of patients with high blood pressure would likely use a repeated-measures design, as the same group of patients is being measured at two different time points and the order of the measurements (before or after medication) is being controlled.

Learn more about the independent-measures design at

https://brainly.com/question/19053601?referrer=searchResults

#SPJ4

Two athletes start from the same point and move on a closed track of 600m. If they run in the same
direction at speeds 20m/s and 30m/s, when will they cross each other?
Mark one or more correct options
(A) 45 sec
(B) 1 min
(C) 1 min 30 sec
(D) 3 min
(E) 5 min

Answers

The solution is Option B , Option D , Option E

The athletes with speeds 20m/s and 30m/s will meet at every minute

What is Speed?

Speed is defined as the rate of change of position of an object in any direction. Speed is measured as the ratio of distance to the time in which the distance was covered. Speed is a scalar quantity as it has only direction and no magnitude

Speed = Distance / Time

Given data ,

Let the equation be represented as A

Now , the value of A is

The distance of the track is D = 600 m

Let the first athlete be A

Let the second athlete be B

The speed of A = 20 m/s

The speed of B = 30 m/s

The time taken by A to complete 600 m = 600 / 20 = 30 seconds

The time taken by B to complete 600 m = 600 / 30 = 20 seconds

Now , the time at which A and B will cross each other is the LCM of their respective time

So , The LCM of 20 and 30 is = 60 seconds

Therefore , the athletes will meet together at every 60 seconds or every 1 minute

Hence , the athletes will meet at every minute

To learn more about speed click :

https://brainly.com/question/19930939

#SPJ1

The fare charged for a rideshare service is a function of
the distance traveled. However, the fare differs
according to the time of day, availability, and other
variables. The distance and fares for 10 rides are
shown in the table. The equation of the least-squares
regression line is ý = 5.20 +2.33x, where y is the
predicted fare and x is the distance.
Distance
(Miles)
1
3
5
8
10
12
15
16
56
Mark this and return
Fare
(Dollars)
1.91
13.68
16.52
24.15
24.79
39.87
40.24
53.84
What is the residual for the rideshare cost with a
distance of 16 miles?
2.33
5.21
11.35
42.49

Answers

Answer: 11.35

Step-by-step explanation: The residual for the rideshare cost with a distance of 16 miles can be calculated using the equation of the least-squares regression line and the observed fare for that distance. The residual is defined as the difference between the observed value and the predicted value.

To find the predicted fare for a distance of 16 miles, we can substitute 16 for x in the equation of the least-squares regression line: ý = 5.20 + 2.33x. This gives us ý = 5.20 + 2.33 * 16 = 42.49.

The observed fare for a distance of 16 miles is 53.84, so the residual is 53.84 - 42.49 = 11.35. Therefore, the answer is 11.35.

Weekly wages at a certain factory are
normally distributed with a mean of
$400 and a standard deviation of $50.
Find the probability that a worker
selected at random makes between
$500 and $550.

Answers

Answer:

The probability that a worker selected at random makes between $500 and $550 is approximately 0.24.

Step-by-step explanation:

To calculate this probability, we can use the standard normal distribution, which is a normal distribution with a mean of 0 and a standard deviation of 1. We can convert the weekly wage amounts to standard units by subtracting the mean and dividing by the standard deviation.

For wages between $500 and $550, the corresponding standard units are:

$500 - $400 = $100 / $50 = 2

$550 - $400 = $150 / $50 = 3

We can use a standard normal table or a calculator to find the probability that a value chosen at random from the standard normal distribution falls within the range of 2 to 3 standard units. This probability is approximately 0.24.

Since the weekly wages at the factory are normally distributed with a mean of $400 and a standard deviation of $50, this probability is also the probability that a worker selected at random makes between $500 and $550.

Maya has 120 caramel apples to sell. Each caramel apple is covered with one topping.

• of the caramel apples are covered with peanuts.
• are covered with chocolate chips.
• are covered with coconut.
• The rest are covered with sprinkles.

How many caramel apples are covered with sprinkles?

Answers

The number of  caramel apples that are covered with sprinkles is; 20

How to solve algebra word problems?

We are given;

Total number of caramel apples to sell = 120

1/5 of the caramel apples are covered with peanuts.

1/3 of the caramel apples are covered with chocolate chips.

3/10 of the caramel apples are covered with coconut.

The rest are covered with sprinkles.

Let 'n' be the number of caramel apples are covered with sprinkles. Thus, we can develop the equation as;

120 = (1/5 * 120) + (1/3 * 120) + (3/10 * 120) + n

120 = 24 + 40 + 36 + n

n = 120 - 100

n = 20

Which means that 20 are those covered with sprinkles

Read more about Algebra Word problems at; https://brainly.com/question/21405634

#SPJ1

Complete question is;

Maya has 120 caramel apples to sell. Each caramel apple is covered with one topping.

1/5 of the caramel apples are covered with peanuts.

1/3 of the caramel apples are covered with chocolate chips.

3/10 of the caramel apples are covered with coconut.

The rest are covered with sprinkles.

How many caramel apples are covered with sprinkles?

X-3y=12 what does it look like on the graph

Answers

The graph of x-3y = 12 is given.

What is graph?

A graph is a visual representation or diagram that displays facts or values in an organized manner in mathematics. The points on a graph are typically used to depict the relationships between two or more things.

The equation is

x-3y = 12.

The graph of this line intersects the x-axis and y-axis.

And end behavior is infinity on both positive and negative.

And the graph is a straight line graph.

Therefore, behavior of x -3y = 12 is given in the graph.

To learn more about the graph;

https://brainly.com/question/23680294

#SPJ1

What rigid motion maps the solid-line figure onto the dotted-line figure?
A reflection
B. rotation
C. translation

Answers

Answer:

A. Reflection

Step-by-step explanation:

I believe the answer would be reflection

You are playing a new video game. The table shows the proportional relationship between the number of levels completed and the time it took you to complete them.


Number of Levels 3 5
Time (hours) ? 2.5


How many minutes does it take you to complete 3 levels?
105 minutes
90 minutes
60 minutes
50 minutes
PLS HELP

Answers

Answer:

the answer is 90 mins option b

Step-by-step explanation:

Answer:

the answer is 90 minutes or B

hope this helps

mark me brainliest

Step-by-step explanation:

Which expression is equivalent to (8×10−2) (2.4×10−3)?

Answers

The expression that is equivalent to the given expression, (8×10−2) (2.4×10−3), is 1.92 × 10⁻⁴

Determining an equivalent expression

From the question, we are to determine the expression that is equivalent to the given expression

From the given information,

The given expression is

(8×10−2) (2.4×10−3)

First, we will write this expression properly

The given expression written properly is

(8 × 10⁻²)(2.4 × 10⁻³)

Now, we will evaluate the expression

Evaluating the expression

(8 × 10⁻²)(2.4 × 10⁻³)

8 × 2.4 × 10⁻² × 10⁻³

19.2 × 10⁻⁵

= 1.92 × 10⁻⁴

Hence, the expression is 1.92 × 10⁻⁴

Learn more on Equivalent expression here: https://brainly.com/question/15775046

#SPJ1


Cómo se determina el condominio de la función cuadrática.

Answers

The domain is the set of all values in the set (- ∞, ∞).

What is a mathematical function, equation and expression?          

Function : In mathematics, a function from a set X to a set Y assigns to each element of X exactly one element of Y. The set X is called the domain of the function and the set Y is called the codomain of the function

Expression : A mathematical expression is made up of terms (constants and variables) separated by mathematical operators

Equation : A mathematical equation is used to equate two expressions.

Given is a quadratic function as follows -

ax² + bx + c

For any quadratic equation (with vertex at origin), the value of [y] or the range is either the set of all positive values from 0 to infinity when the quadratic function open upwards and  the set of all negative values from 0 to -infinity when the quadratic function open downwards. The domain is the set of all values in the set (- ∞, ∞).

Therefore, the domain is the set of all values in the set (- ∞, ∞).

To solve more questions on functions, expressions and polynomials, visit the link below -

brainly.com/question/17421223

#SPJ1

{Question in english language is as follows -

How the domain of the quadratic function is determined.}

If the slope of the regression line is calculated to be 2.5 and the intercept 16 then the value of Y when X is 4 is:
Select one or more:
a.2.5
b.66.5
c.26
d.16

Answers

In order to calculate linear regression, we apply the formula Y = B + X. The current access level for 16 plus is 2.5. As a result, Option A is the right response. We get 26 from 16 + 10.

How do you interpret the slope and intercept of a regression line?

The slope and intercept of a line show how steep it is, and the intercept shows where the line crosses an axis. It is possible to calculate an average rate of change using the slope and intercept, which describe the linear relationship between two variables.

An equation of the form Y = a + bX, where X is the explanatory variable and Y is the dependent variable, represents a linear regression line. A line's intercept (the value of y when x = 0) is equal to the slope of the line, which is b.

The regression slope intercept equation, [tex]y' = b_{0} + b_{1} x[/tex], where "[tex]b_{0}[/tex]" is the y-intercept and "[tex]b_{1} x[/tex]" is the slope, is actually just an algebraic version of the regression equation. Once the equation for linear regression has been discovered,

Therefore the correct answer is option a ) 2.5

To learn more about regression refer to :

https://brainly.com/question/25987747

#SPJ4

Question 3 (2 points)
For the following, find the arc length:
(Use π=3.14 and round your final answer to the hundredths.)

Answers

The length of the arc is 10.47 units

How to determine the length of the arc?

From the question, we have the following parameters that can be used in our computation:

Radius = 10

Angle = π/3

The length of the arc can be calculated using

Arc length = Radius * Angle

Substitute the known values in the above equation, so, we have the following representation

Arc length = 10 * π/3

Evaluate

Arc length = 10.47

Hence, the arc length is 10.47

Read more about arc length at

https://brainly.com/question/16552139

#SPJ1

write the prime factorization the foll numbers
A.144​

Answers

Answer: 2 times 2 times 2 times 2 times 3 times 3

Step-by-step explanation:

Determine whether the sequence converges or diverges. If it converges, find the limit. (If an answer does not exist, enter DNE.)
an =
n sqr root 35 + 3n
lim nâ[infinity] an =

Answers

This sequence is not convergent because the nth term does not approach a single limit as n approaches infinity. The limit does not exist, so the answer is DNE.

This sequence does not converge to a single limit because the nth term of the sequence, an, has a coefficient of n, which means that the value of an increases without bound as n increases. This means that the value of an does not approach a single limit as n approaches infinity, and so the limit does not exist. Therefore, the answer for the limit as n approaches infinity is DNE.

This sequence does not converge to a single limit because the nth term of the sequence, an, has a coefficient of n, which means that the value of an increases without bound as n increases. As n increases, the value of an increases, and so the value of an does not approach a single limit as n approaches infinity. This means that the limit does not exist, and so the answer for the limit as n approaches infinity is DNE.

Learn more about convergence here

https://brainly.com/question/15415793

#SPJ4

Suppose two dice are rolled. Let X be the random variable measuring the sum of the two numbers rolled.
(a) Find the probability mass function for X.
(b) Find the expected value E(X).
(c) Find the variance V(X).

Answers

The expected value E(X) is 7 and the value of variance of X is 5.8333.

Two dice are rolled. Let X be the random variable measuring the sum of the two numbers rolled.

a) The probability of mass function is obtained below:

The possible outcomes in each of the dice are 1 to 6. Therefore, the possible outcomes when two dice is rolled is 36

The sample space, s for fair dice (red die and blue die) is given below:

N(s) = 36

From the given information, two dice are rolled let X be the random variable measuring the sum of the two numbers rolled.

b) The expected value is calculated below:

The probability mass function of X is,

The required mean is,

E(x) = ∑xP(X=x)

[tex]=[2[/tex]×[tex]\frac{1}{36}+3[/tex]×[tex]\frac{2}{36}[/tex]+....+11×[tex]\frac{2}{36}+12[/tex]×[tex](\frac{1}{36} )[/tex]]

=[0.0556+0.1667+0.3333+0.5556+0.8333+1.1111+1.000+0.8333+0.6111+0.3333

=7

C) The variance V(X) is calculated below:

The probability mass function of X is,

The required variance is,

V(x)= ∑[tex](X-x)^{2}P(x) = 5.8333[/tex]

Therefore, the expected value E(X) is 7 and the value of variance of X is 5.8333.

For such more questions about probability:

https://brainly.com/question/99675

#SPJ4

Use Green's Theorem to evaluate the line integral along the given positively oriented curve.
integral.gif C (3y + 7e^sqrt(x)) dx + (8x + 5 cos y^2) dy
C is the boundary of the region enclosed by the parabolas y = x2 and x = y2

Answers

On solving the provided question, From the question, our region is defined by: lower bound: y= [tex]x^2[/tex] and upper bound: y = [tex]\sqrt{x}[/tex]

what is integration?

Integrals are mathematical representations of numbers and functions that express notions such as volume, area, displacement, and other outcomes of the combination of little data. Finding integrals is the term used to describe the procedure.

By green's theorem -

[tex]\int\limits^{}_{} {} \, \int\limits^{}_{a} {5dA} \, = 5/3[/tex]

First, the integral given in this exercise corresponds to:  

[tex]\int\limits^{}_{C} {((3y + 7e\sqrt{x}dx) +( 8x+ 5cos(y^2)) } \, dy[/tex]

Greens Theorem given as,

[tex]\int\limits^{}_{C} {(P(x,y)dx +Q(x,y)dy)} \, = \int\limits^{}_{} {x} \, \int\limits^{}_a {(-\beta /\beta _{y} )P(x,y) + (\beta /\beta _{y} )Q(x,y)} \, dA[/tex]

and we have -

P(x,y) = 3y + [tex]7e^{\sqrt{x}}[/tex]

Q(x,y) = 8x + 5cos([tex]y^2[/tex])

And,

[tex]\int\limits^{}_{C} {(P(x,y)dx +Q(x,y)dy)} \, = \int\limits^{}_{} {x} \, \int\limits^{}_a {5dA} \, dA[/tex]

From the question, our region is defined by:

lower bound: y= [tex]x^2[/tex]

upper bound: y = [tex]\sqrt{x}[/tex]

To know more about integration visit:
https://brainly.com/question/18125359

#SPJ4

Which points are included in the graph of the ceiling function f(x) = ⌈x⌉? Select all that apply.
A. (−0.5, −1)
B. (−1.5, −2)
C. (−3.5, −3)
D. (−3.5, −4)
E. (3.001, 4)
F. (−3.001, −3)

Answers

Answer:

  C, E, F

Step-by-step explanation:

You want to know which points match the ceiling function among ...

A. (−0.5, −1)B. (−1.5, −2)C. (−3.5, −3)D. (−3.5, −4)E. (3.001, 4)F. (−3.001, −3)

Ceiling

The ceiling function is also known as the least integer function. It returns the smallest integer greater than or equal to its argument value. It is essentially a "round to next higher" function, with any integer input remaining unchanged.

The attachment shows the input and output of the ceiling function for the values on the answer choice list. C, E, F are points on the function's graph.

Business Organization

1. James invests $10,000 in a partnership with 3 other people. One of those people also invested

$10,000 and the other two invested $90,000 each. What percent of the business does James

own?



(please help me if I don't get help I won't graduate)

Answers

James owns just 5% of the company after the investment.

What is a percentage?

Percentage can be calculated by dividing the value by the total value, and then multiplying the result by 100.

Percentage is a number or ratio expressed as a fraction of 100. It is often denoted using the percent sign, "%".

Percentage can be calculated by dividing the value by the total value, and then multiplying the result by 100. It is given by:

Percentage = (value / total value) * 100%

How to find the Percentage owned by James;

James' investment: $10,000

The other 3 people invested;

x = $10,000

y = $10,000

z = $10,000

Total investment made was = $200,000

James' Percentage = [tex]\frac{10000}{200000\\}[/tex] x 100

James' Percentage = 5%

So, we can say that James owns 5% of the total investment in the business.

Learn more on percentages here: https://brainly.com/question/24877689

#SPJ1

I need help with the word problem

Answers

Answer:

BAG 1 IS BETTER. IT IS BETTER TO PAY $18 FOR 12 POUNDS OF CAT FOOD.

Step-by-step explanation:

To find the unit cost, we need to know how much one pound of cat food cost for each bag

Bag 1

It said you get 12 pounds for $18

Divide $18 by 12 to find how much ONE pound is.

18 ÷ 12 = 1.5

ONE POUND COST   $1.5  

This is the unit cost

Let's do the same for the other bag of cat food!

Bag 2

Divide $24 by 15

24 ÷ 15 = 1.6

ONE POUND COST   $1.6

This is the unit cost

It is better to pay for $1.5 per pound because it is LESS than $1.6

You will save money this way.

BAG 1 IS BETTER. IT IS BETTER TO PAY $18 FOR 12 POUNDS OF CAT FOOD.

which angle corresponds with 3

Answers

∠8 is the corresponds with ∠3.

Hence, option B is the correct answer.

What is Corresponding pairs ?

Congruent angles are those that correspond. Corresponding pairs are all angles that are positioned in relation to the parallel and transversal lines in the same way. When two parallel lines are intersected by another line, comparable angles are the angles that are created in matching corners or corresponding corners with the transversal (i.e. the transversal).

we know that

m∠3=m∠8 --------> by corresponding angles

m∠7=m∠8 -------> by vertical angles

so

m∠3=m∠7 -------> by alternate interior angles

Learn more about the Corresponds Angle here: https://brainly.com/question/22190474

#SPJ1

NOTE: The given question is incomplete on the portal. Here is the complete question.

QUESTION: Which angle corresponds to ∠3?

A) ∠6

B) ∠8

C) ∠1

D) ∠7

Suppose previous research suggests that the mean length of all adult Anacondas is 13 feet with a standard deviation of 2.4 feet. Let W be the random variable that stands for length of adult Anacondas, so E(W)=13, SD(W)=2.4. You are planning on collecting a random sample of 50 adult Anacondas. Consider the RV Bar-W, which is the mean of the 50 sampled Anacondas. 98% of samples will have the realized value of Bar-W less than what value? Which of the answers reasonably approximates the requested value of the sample mean with justification?a. "Bar-w"=14.67 since we can use a normal approximation by the CLTb. "Bar-w"=17.93 since we can use a normal approximation by the CLTc. "Bar-w"=13.70 since we can use a normal approximation by the CLTd. "Bar-w"=12.30 since we can use a normal approximation by the CLTe.A normal approximation is inappropriate

Answers

The answers reasonably approximates the requested value of the sample mean with justification is c. "Bar-w"=13.70 since we can use a normal approximation by the CLT.

Since the sample size is large (n=50), we can use the central limit theorem (CLT) to approximate the sampling distribution of the sample mean with a normal distribution.

The mean of the sampling distribution of the sample mean is equal to the mean of the population, which is 13 feet in this case. The standard deviation of the sampling distribution of the sample mean is equal to the standard deviation of the population divided by the square root of the sample size. This is known as the standard error of the mean, and is denoted by SE(Bar-W).

In this case, SE(Bar-W) = 2.4/sqrt(50) = 0.48.

To find the 98th percentile of a normal distribution with mean 13 and standard deviation 0.48, we can use a standard normal table or a calculator to find that the 98th percentile is approximately 2.05.

Therefore, the value that 98% of samples will have the realized value of Bar-W less than is approximately 13 + 2.05 * 0.48 = 13.70. This means that the correct answer is (c) "Bar-w"=13.70 since we can use a normal approximation by the CLT.

Learn more about sample, here https://brainly.com/question/11045407

#SPJ4

find the maximum area of the rectangle inscribed in the triangle formed by the x x-axis, the y y-axis and the line y

Answers

A shape's highest possible area is called its maximum area.The size of the rectangle that will result in the greatest area is 4 by 2 units.

What is meant by rectangle?Four sides, four corners, and four 90° right angles make up the closed, 2-D shape of a rectangle. A rectangle has parallel, equal sides on either side. Rectangles are two-dimensional shapes, and as such, they have length and breadth as their defining characteristics. The rectangle's shorter side is its width, while its longer side is its length.How to calculate a rectangle's area. The size of a rectangle. A = l × b. Once the length and breadth are known for any rectangle, the area may be determined. The area of the rectangle is calculated as a square-unit dimension by multiplying length and width.

The equation is given as:

x + 2y - 8 = 0

Rewrite as:

2y = 8 - x

Divide both sides by 2,

y =  [tex]\frac{8 - x}{2}[/tex]

y = 4 - [tex]\frac{x}{2}[/tex]

The area (A) of the rectangle is:

A = xy

Substitute

y = 4 - [tex]\frac{x}{2}[/tex]

A = x[tex](4 - \frac{x}{2})[/tex]

Expand

A = 4x - [tex]\frac{x^{2} }{2}[/tex]

Differentiate

A' = 4 - x

Set the derivative to 0,

4 - x = 0

Collect like terms

x = 4

Recall that:

y = 4 - [tex]\frac{x}{2}[/tex]

Substitute 4 for x,

y = 4 - [tex]\frac{4}{2}[/tex]

y = 4 - 2

y = 2

So, we have:

x = 4 and y = 2

Hence, the dimension of the rectangle that will produce the maximum area is 4 units by 2 units.

The complete question is:

A rectangle is inscribed in the region bounded by the x-axis, the y-axis, and the graph of x+2y-8=0. Approximate the dimensions of the rectangle that will produce the maximum area.

To learn more about rectangle refer to:

brainly.com/question/25292087

#SPJ4

f the temperature is 50 degrees Fahrenheit, what is the temperature in Celsius?

Answers

Answer:

the answer is 10 degrees Celsius

50 Degrees Fahrenheit is 10 degrees Celsius.

Here is the formula

(50°F − 32) × 5/9 = 10°C

For the function f(x)  x  9, find (f  f 1)(5)

Answers

the value of [tex]f o f^{-1}[/tex](5) of the function f(x) = x + 9 is 5

What is the inverse of a function?

An inverse function or an anti function is defined as a function, which can reverse into another function. In simple words, if any function “f” takes x to y then, the inverse of “f” will take y to x. If the function is denoted by ‘f’ or ‘F’, then the inverse function is denoted by f^-1 or F^-1. One should not confuse (-1) with exponent or reciprocal here.

[tex]f^{-1}[/tex](x) =

if f(x) = x + 9

setting x + 9 = y and making x the subject we have

y = x + 9

x = y - 9

replacing x with [tex]f^{-1}[/tex](x) and y with x we have

[tex]f^{-1}[/tex](x) = x - 9

so the f o [tex]f^{-1}[/tex](x) can be gotten by substituting x in f(x) with x - 9 so that we have

f o [tex]f^{-1}[/tex](x) = (x - 9) + 9

f o [tex]f^{-1}[/tex](x) = x

so f o [tex]f^{-1}[/tex](5) will be x = 5

which becomes

f o [tex]f^{-1}[/tex](5) = 5

In conclusion f o [tex]f^{-1}[/tex](x) is 5

Learn more about inverse function: https://brainly.com/question/1942556

#SPJ1

suppose 2n c 1 numbers are selected from f1; 2; 3; : : : ; 4ng. using the pigeonhole principle, show that there must be two selected numbers whose difference is 2. clearly indicate what are the pigeons, holes, and rules for assigning a pigeon to a hole.

Answers

Yes, there must be two selected numbers whose difference is 2. Any assignment of n+1 pigeons to n holes. Then there exists at least one hole with at least 2 pigeons in it.

The Pigeon-Hole Principle :

A general rule, called the pigeon -hole principle, is that if there are more pigeons than holes, there should be at least one hole with at least two pigeons. If we put n + 1 objects into n boxes, we can say that at least one box contains 2 or more objects. Example: If there are Kn+ 1 (where k is a positive integer) pigeons distributed in n holes, then the holes contain at least k + 1 pigeons. Consider 2n pair of pigeon holes (1,3), (2,4), (5,7),(6,8), .. ..,(4n-3,4n-1),(4n-2,4n). The 2 numbers in each pair have the difference 2.

If we choose 2n+1 numbers(pigeons), then they cannot all belong to different pairs.

Thus by pigeon hole principle, there must be two selected numbers whose difference is 2.

To learn more about Pigeonhole principle, refer:

https://brainly.com/question/13982786

#SPJ4

Other Questions
Which percentage of DNA would indicate the organisms are more closely related?A. 30%B. 10%C. 20%D. 95% Identify the true statement. Choose one: A. The troposphere is the highest layer within the homosphere and sits below the mesopause. B. The mesosphere is ozone-rich and convects as warm air rises and cold air sinks. C. The thermosphere has distinct layers of gases of different densities and therefore is called the heterosphere. D. Ozone is a trace gas that protects the Earth's surface from excessive infrared radiation. 3 consecutive even numbers that add up to 3000 Two pea plants heterozygous for the characters of pod color and pod shape are crossed. Assume that G allele stands for green pod color, g allelefor yellow pod color, I allelefor inflated pod shape, and i allelefor constricted pod shape. Complete the Punnett square for this cross. What is the molarity of a solution made with 87g of Mgo dissolved in enough water to make800mL of solution? if you had a chance to meet and hang out with one celebrity and be friends with them who would it be? Pls I need to know!! How to find the net ionic of an equation Mr. Frank has 1.08 kilograms of fertilizer for the plants in his nursery. He wants every plant to get 93 mg of fertilizer 4 times each year. What is the number of plants he could fertilize with that amount? How much fertilizer will he have left over? Please help me with this problem!! find the gradient of the line AB What is the product of -8 and -7 Plz help What factors determinehow the speed of the marbles changes in acollision? Is the group of words in bold a phrase or a clause?The malleus, incus, and stapes-also called the hammer, anvil, and stirrup- that help transmit sound vibrations. find the vertex h(r)=r^(2)+11r-26 PLEASE HELP ME FIGURE THIS OUT help me please and answer correctly people answer wrong on purpose The Table shows the relationship and speeds between a cheetah and a line. What is the missing value in the table?Cheetah (miles per hour)7 14 ? 28 35Lion (miles per hour)5 10 15 20 25A. 19B. 21C. 23D. 25Please help ASAP!! Since the conflict of the novel has already been established, what part of the plot is going on now? In the outsiders The vertices of a triangle MNP are M(5,6) , N(0,3) , and P(4,-2). What is the coordinate of the midpoint of the line connecting the midpoints of sides MN and MP 16 gallons of water are used to completely fill 5 fish tanks. If each tank holds the same amount of water, how many gallons will each tank hold?